nmop_apisdn2
Thanks Received: 16
Vinny Gambini
Vinny Gambini
 
Posts: 24
Joined: June 23rd, 2012
 
 
 

Q8 - Political theorist: The chief foundations

by nmop_apisdn2 Sat Jul 14, 2012 10:20 pm

This question is a flaw question, and they're my favorite type to answer. I mean, LSAT questions are FULL of flaws, so why not have fun doing them! I digress...

So in this question, right off the bat I noticed that there was a conditional statement:

"as there cannot be a good legal system where the police are not well paid"

Logically, this can be interpreted as "If the police are not well-paid, then you cannot have a good legal system". Don't be afraid to rethink the conditional statement; the LSAT loves to try and hide the conditionals, for that person who was not paying very much attention during the conditional logic chapter in their book.

You can diagram this as:
~WP --> ~GLS

WP= Well paid and GLS= Good legal system

(the ~ symbol denotes a negation)

The contrapositive of this statement would be:

GLS --> WP

Okay, so where is the flaw exactly, you might ask?

Well, in the last sentence of the stimulus, the conclusion, the political theorist states "it follows that where the police are well paid there will be a good legal system".

AHA! Right off the bat, I see that the political theorist is saying that if the police are well paid (WP), then the good legal system (GLS) must follow. But as you'll see, by looking at our contrapositive statement, well paid (WP) police officers is a necessary condition to the good legal system (GLS). This type of fallacy is known as affirming the consequent (the necessary condition), which if logically done, will never logically lead to affirming the antecedent (sufficient condition).This is sometimes known as the converse error. (click link to see this explained in more detail)

Question: The reasoning in the argument is not sound because it fails to establish that:

Answer choices:

(A) is incorrect because the political theorist does not fail to establish that many governments with bad legal systems have poorly paid officers. Heck, we don't need to know anything about other governments in order to analyze the logic of this stimulus. Therefore, this answer choice is out of scope.
(B) is incorrect because if you diagram it, you get:

GLS --> ~WP, which is logically inconsistent with our diagrammed analysis of the premises or conclusion; so get rid of it.

(C) is incorrect because this is talking about the effectiveness of a well paid police force, which the argument in the stimulus never discussed; therefore, this answer choice is out of scope.
(E) is incorrect for the same reason that (A) was incorrect; the argument does not need to establish that some bad governments have good legal systems because frankly, this has nothing to do with the logic of the argument.

Lastly, answer choice (D) is correct because the argument fails to establish that a well paid police force is sufficient go guarantee a good legal system, because as far as we can tell, being well-paid (WP) is necessary to establishing a good legal system (GLS), but not sufficient. Therefore, the argument fails to show us that its sufficient as well.

Eeek, I guess I exaggerated when I said that this was fun! lol
User avatar
 
maryadkins
Thanks Received: 641
Atticus Finch
Atticus Finch
 
Posts: 1261
Joined: March 23rd, 2011
 
 
 

Re: Q8 - Political theorist: The chief foundations

by maryadkins Fri Jul 20, 2012 3:47 pm

Nicely explained! Thanks so much for this!
 
syousif3
Thanks Received: 0
Jackie Chiles
Jackie Chiles
 
Posts: 36
Joined: July 19th, 2012
 
 
 

Re: Q8 - Political theorist: The chief foundations

by syousif3 Thu Aug 16, 2012 5:34 pm

I'm confused with your last statement, what do you mean by "Therefore, the argument fails to show us that its sufficient as well" .. It can never be like this (WP->GLS) though. I'm also confused with the wording of the question.. "....... fails to establish that " is it the same as saying it mistakenly establishes that WP is sufficient?
 
timmydoeslsat
Thanks Received: 887
Atticus Finch
Atticus Finch
 
Posts: 1136
Joined: June 20th, 2011
 
 
trophy
Most Thanked
trophy
First Responder
 

Re: Q8 - Political theorist: The chief foundations

by timmydoeslsat Fri Aug 17, 2012 5:40 pm

Its important to know that we have a question stem that is no different than a flaw question stem. We are just showing why it is flawed. To show how something is flawed, we can expect to show why the evidence does not fully support the conclusion reached.

In this argument, we know:

Government ---> LS and Police (These are the chief foundations)
~Police Well Paid---> ~Good LS
____________________________
Police Well Paid ---> Good LS


The evidence given has not shown that simply paying the police well is enough to give us a Good LS. The evidence tells us that having a Good LS requires Police Well Paid.

This argument, as is the case with all invalid arguments, fail to establish their conclusions as true. You can rephrase this is many ways, such as that this argument fails to consider that the police being well paid can also lead to ~Good LS.
 
tara_amber1
Thanks Received: 5
Jackie Chiles
Jackie Chiles
 
Posts: 29
Joined: August 15th, 2014
 
 
 

Re: Q8 - Political theorist: The chief foundations

by tara_amber1 Sat Sep 13, 2014 4:59 pm

I got this answer right because I spotted there was a problem with the sufficient/necessary condition in the stimulus. However, the prompt says "...because it fails to establish that," so wouldn't it make sense for the correct answer choice to read "...a well paid police force is necessary to guarantee a good legal system"?

I was confused by the answer choice because the diagrams in previous posts, and my diagram both have the contrapositive of the first statement as

GLS --- > WP

with well paid as a necessary condition. And the argument's fallacious last statement says the converse of this

WP ----> GLS

So, wouldn't the answer be true if the prompt read: "what is the flaw in the stimulus?" instead of saying would it failed to achieve? Because it already achieved that the WP was in the sufficient position.

:?